Stay ahead of learning milestones! Enroll in a class over the summer!

G
Topic
First Poster
Last Poster
Inspired by hlminh
sqing   2
N an hour ago by SPQ
Source: Own
Let $ a,b,c $ be real numbers such that $ a^2+b^2+c^2=1. $ Prove that $$ |a-kb|+|b-kc|+|c-ka|\leq \sqrt{3k^2+2k+3}$$Where $ k\geq 0 . $
2 replies
1 viewing
sqing
Yesterday at 4:43 AM
SPQ
an hour ago
A cyclic inequality
KhuongTrang   3
N an hour ago by KhuongTrang
Source: own-CRUX
IMAGE
https://cms.math.ca/.../uploads/2025/04/Wholeissue_51_4.pdf
3 replies
KhuongTrang
Monday at 4:18 PM
KhuongTrang
an hour ago
Tiling rectangle with smaller rectangles.
MarkBcc168   60
N 2 hours ago by cursed_tangent1434
Source: IMO Shortlist 2017 C1
A rectangle $\mathcal{R}$ with odd integer side lengths is divided into small rectangles with integer side lengths. Prove that there is at least one among the small rectangles whose distances from the four sides of $\mathcal{R}$ are either all odd or all even.

Proposed by Jeck Lim, Singapore
60 replies
MarkBcc168
Jul 10, 2018
cursed_tangent1434
2 hours ago
ALGEBRA INEQUALITY
Tony_stark0094   2
N 2 hours ago by Sedro
$a,b,c > 0$ Prove that $$\frac{a^2+bc}{b+c} + \frac{b^2+ac}{a+c} + \frac {c^2 + ab}{a+b} \geq a+b+c$$
2 replies
Tony_stark0094
3 hours ago
Sedro
2 hours ago
Checking a summand property for integers sufficiently large.
DinDean   2
N 2 hours ago by DinDean
For any fixed integer $m\geqslant 2$, prove that there exists a positive integer $f(m)$, such that for any integer $n\geqslant f(m)$, $n$ can be expressed by a sum of positive integers $a_i$'s as
\[n=a_1+a_2+\dots+a_m,\]where $a_1\mid a_2$, $a_2\mid a_3$, $\dots$, $a_{m-1}\mid a_m$ and $1\leqslant a_1<a_2<\dots<a_m$.
2 replies
DinDean
Yesterday at 5:21 PM
DinDean
2 hours ago
Bunnies hopping around in circles
popcorn1   22
N 2 hours ago by awesomeming327.
Source: USA December TST for IMO 2023, Problem 1 and USA TST for EGMO 2023, Problem 1
There are $2022$ equally spaced points on a circular track $\gamma$ of circumference $2022$. The points are labeled $A_1, A_2, \ldots, A_{2022}$ in some order, each label used once. Initially, Bunbun the Bunny begins at $A_1$. She hops along $\gamma$ from $A_1$ to $A_2$, then from $A_2$ to $A_3$, until she reaches $A_{2022}$, after which she hops back to $A_1$. When hopping from $P$ to $Q$, she always hops along the shorter of the two arcs $\widehat{PQ}$ of $\gamma$; if $\overline{PQ}$ is a diameter of $\gamma$, she moves along either semicircle.

Determine the maximal possible sum of the lengths of the $2022$ arcs which Bunbun traveled, over all possible labellings of the $2022$ points.

Kevin Cong
22 replies
popcorn1
Dec 12, 2022
awesomeming327.
2 hours ago
Iran second round 2025-q1
mohsen   4
N 2 hours ago by MathLuis
Find all positive integers n>2 such that sum of n and any of its prime divisors is a perfect square.
4 replies
mohsen
Apr 19, 2025
MathLuis
2 hours ago
Dear Sqing: So Many Inequalities...
hashtagmath   37
N 3 hours ago by hashtagmath
I have noticed thousands upon thousands of inequalities that you have posted to HSO and was wondering where you get the inspiration, imagination, and even the validation that such inequalities are true? Also, what do you find particularly appealing and important about specifically inequalities rather than other branches of mathematics? Thank you :)
37 replies
hashtagmath
Oct 30, 2024
hashtagmath
3 hours ago
integer functional equation
ABCDE   148
N 3 hours ago by Jakjjdm
Source: 2015 IMO Shortlist A2
Determine all functions $f:\mathbb{Z}\rightarrow\mathbb{Z}$ with the property that \[f(x-f(y))=f(f(x))-f(y)-1\]holds for all $x,y\in\mathbb{Z}$.
148 replies
ABCDE
Jul 7, 2016
Jakjjdm
3 hours ago
IMO Shortlist 2013, Number Theory #1
lyukhson   152
N 3 hours ago by Jakjjdm
Source: IMO Shortlist 2013, Number Theory #1
Let $\mathbb{Z} _{>0}$ be the set of positive integers. Find all functions $f: \mathbb{Z} _{>0}\rightarrow \mathbb{Z} _{>0}$ such that
\[ m^2 + f(n) \mid mf(m) +n \]
for all positive integers $m$ and $n$.
152 replies
lyukhson
Jul 10, 2014
Jakjjdm
3 hours ago
IMO Shortlist 2011, Algebra 7
orl   23
N Apr 15, 2025 by bin_sherlo
Source: IMO Shortlist 2011, Algebra 7
Let $a,b$ and $c$ be positive real numbers satisfying $\min(a+b,b+c,c+a) > \sqrt{2}$ and $a^2+b^2+c^2=3.$ Prove that

\[\frac{a}{(b+c-a)^2} + \frac{b}{(c+a-b)^2} + \frac{c}{(a+b-c)^2} \geq \frac{3}{(abc)^2}.\]

Proposed by Titu Andreescu, Saudi Arabia
23 replies
orl
Jul 11, 2012
bin_sherlo
Apr 15, 2025
IMO Shortlist 2011, Algebra 7
G H J
G H BBookmark kLocked kLocked NReply
Source: IMO Shortlist 2011, Algebra 7
The post below has been deleted. Click to close.
This post has been deleted. Click here to see post.
orl
3647 posts
#1 • 12 Y
Y by tastymath75025, tritanngo99, Adventure10, Mango247, Rounak_iitr, ehuseyinyigit, cubres, and 5 other users
Let $a,b$ and $c$ be positive real numbers satisfying $\min(a+b,b+c,c+a) > \sqrt{2}$ and $a^2+b^2+c^2=3.$ Prove that

\[\frac{a}{(b+c-a)^2} + \frac{b}{(c+a-b)^2} + \frac{c}{(a+b-c)^2} \geq \frac{3}{(abc)^2}.\]

Proposed by Titu Andreescu, Saudi Arabia
This post has been edited 2 times. Last edited by orl, Jul 24, 2012, 1:35 PM
Z K Y
The post below has been deleted. Click to close.
This post has been deleted. Click here to see post.
oneplusone
1459 posts
#2 • 35 Y
Y by Nguyenhuyhoang, huyvietnam, sayantanchakraborty, MathPanda1, AdBondEvent, quangminhltv99, JasperL, AlgebraFC, yiwen, Wizard_32, abdelkrim, HolyMath, karitoshi, Wizard0001, srijonrick, megarnie, myh2910, ThisNameIsNotAvailable, qwedsazxc, WinterSecret, Adventure10, Mango247, ehuseyinyigit, Rayanelba, farhad.fritl, and 10 other users
It is easy to check that $a+b-c>0$ and its cyclic counterparts. Then by Holder's,
\[\sum_{cyc}{\frac{a}{(b+c-a)^2}}\sum_{cyc}a^2(b+c-a)\sum_{cyc}a^3(b+c-a)\geq (\sum_{cyc}a^2)^3=27\]
By Schurs,
\[\sum_{cyc}a^2(b+c-a)\leq 3abc
\] and
\[\sum_{cyc}a^3(b+c-a)\leq abc(a+b+c)
\] Therefore
\[\sum_{cyc}{\frac{a}{(b+c-a)^2}}\geq \frac{9}{(abc)^2(a+b+c)}\geq\frac{3}{(abc)^2}\]
Z K Y
The post below has been deleted. Click to close.
This post has been deleted. Click here to see post.
daniel73
253 posts
#3 • 7 Y
Y by yshk, Adventure10, Mango247, and 4 other users
For those who do not quickly see why $b+c-a>0$:

Click to reveal hidden text

Alternative solution without Holder's inequality:

Click to reveal hidden text
Z K Y
The post below has been deleted. Click to close.
This post has been deleted. Click here to see post.
mathbuzz
803 posts
#4 • 4 Y
Y by ehuseyinyigit, Adventure10, Mango247, and 1 other user
for solving it , we first prove a claim
claim--- let x,y,z be 3 positive reals. they satisfy $min(x+y,y+z,z+x)> 2^{1/2}$ and $x^2+y^2+z^2=3$.
then x,y,z are the sides of a triangle.
proof--- here we clearly assume an ordering $x >= y >= z$.
then clearly min(x+y,y+z,z+x)=y+z. then y+z > $ 2^{1/2}$. then 2$(y^2+z^2)$ >= $(z+y)^2$ . so, $y^2+z^2$ >$1$. now , clearly , $y+z >x$ [if not , assume that $y+z <= x $. then $x >= y+z > 2^{1/2}$ . so, $x^2 >2$ , which contradicts $x^2+y^2+z^2$=3]. so , x,y,z are the sides of a triangle.

now , applying it to our problem , we get that , a,b,c are the sides of a triangle.so , put a=p+q , b=q+r , c=r+p.
then the given condition translates into the constraint
$2(p^2+q^2+r^2+pq+qr+rp)$=3 and then applying LM method is sufficient . [here we must keep in mind the condition min(p+q,q+r,r+p) >$ sqrt.2$ for choosing
the values of p,q,r for minimizing the expression (if required)]
. the minimum value is at p=q=r=1/2 [calculation is tedious!] :lol:

Click to reveal hidden text
This post has been edited 1 time. Last edited by mathbuzz, Aug 13, 2012, 12:15 PM
Z K Y
The post below has been deleted. Click to close.
This post has been deleted. Click here to see post.
mahanmath
1354 posts
#5 • 8 Y
Y by bvdsf, Adventure10, Mango247, and 5 other users
We had got this inequality at an exam :D , This is what I wrote that time :
Click to reveal hidden text
Z K Y
The post below has been deleted. Click to close.
This post has been deleted. Click here to see post.
Anzoteh
126 posts
#6 • 4 Y
Y by Adventure10, Mango247, and 2 other users
Here is mine. (Outline)
1. Show that $a, b, c$ are sides of triangle, so that $\exists x, y, z\in\mathbb R^{+}$ with $a=x+y, b=x+z, c=y+z$ (which is shown by someone else).
2. The inequality becomes $[x^{2}y^{2}(x+y)+y^{2}z^{2}(y+z)+z^{2}x^{2}(z+x)]\ge 12x^{2}y^{2}z^{2}$
3. Show that $x+y+z\le\frac{3}{2},$ and homogenize the expression in (2) by multiplying LHS by $\frac{2}{3} (x+y+z)$ and RHS by $\frac {1}{27} ((x+y)^{2}+(y+z)^{2}+(z+x)^{2})^{3}.$
4. Expand (3) and we only need to use Muirhead to settle everything (See below)!

So it would be: (all symmetric sum)
$(3\sum x^{8}y^{4}z^{0}+6\sum x^{8}y^{3}z^{1}+3\sum x^{8}y^{2}z^{2}+12\sum x^{7}y^{5}z^{0}+33\sum x^{7}y^{4}z^{1}+39\sum x^{7}y^{3}z^{2}+9\sum x^{6}y^{6}z^{0}+69\sum x^{6}y^{5}z^{1}+102\sum x^{6}y^{4}z^{2}+51\sum x^{6}y^{3}z^{3}+69\sum x^{5}y^{5}z^{2}+153\sum x^{5}y^{4}z^{3}+27\sum x^{4}y^{4}z^{4})\ge (8\sum x^{8}y^{2}z^{2}+48\sum x^{7}y^{3}z^{2}+96\sum x^{6}y^{4}z^{2}+72\sum x^{6}y^{3}z^{3}+56\sum x^{5}y^{5}z^{2}+240\sum x^{5}y^{4}z^{3}+56\sum x^{4}y^{4}z^{4})$
which is obvious by Muirhead

It would be incredible that an A7 problem can be solved by expanding alone.
Z K Y
The post below has been deleted. Click to close.
This post has been deleted. Click here to see post.
utkarshgupta
2280 posts
#7 • 2 Y
Y by Adventure10 and 1 other user
Here will
$\prod (a+b-c) \ge 1$ ?
Z K Y
The post below has been deleted. Click to close.
This post has been deleted. Click here to see post.
Abubakir
68 posts
#8 • 2 Y
Y by Adventure10, Mango247
mathbuzz wrote:
for solving it , we first prove a claim
claim--- let x,y,z be 3 positive reals. they satisfy $min(x+y,y+z,z+x)> 2^{1/2}$ and $x^2+y^2+z^2=3$.
then x,y,z are the sides of a triangle.
proof--- here we clearly assume an ordering $x >= y >= z$.
then clearly min(x+y,y+z,z+x)=y+z. then y+z > $ 2^{1/2}$. then 2$(y^2+z^2)$ >= $(z+y)^2$ . so, $y^2+z^2$ >$1$. now , clearly , $y+z >x$ [if not , assume that $y+z <= x $. then $x >= y+z > 2^{1/2}$ . so, $x^2 >2$ , which contradicts $x^2+y^2+z^2$=3]. so , x,y,z are the sides of a triangle.

now , applying it to our problem , we get that , a,b,c are the sides of a triangle.so , put a=p+q , b=q+r , c=r+p.
then the given condition translates into the constraint
$2(p^2+q^2+r^2+pq+qr+rp)$=3 and then applying LM method is sufficient . [here we must keep in mind the condition min(p+q,q+r,r+p) >$ sqrt.2$ for choosing
the values of p,q,r for minimizing the expression (if required)]
. the minimum value is at p=q=r=1/2 [calculation is tedious!] :lol:

Click to reveal hidden text

What does abbreviation "LM" mean?
Z K Y
The post below has been deleted. Click to close.
This post has been deleted. Click here to see post.
vjdjmathaddict
502 posts
#9 • 1 Y
Y by Adventure10
lagrange multiplier
Z K Y
The post below has been deleted. Click to close.
This post has been deleted. Click here to see post.
ahmedAbd
91 posts
#10 • 4 Y
Y by Wizard_32, kiyoras_2001, Adventure10, Mango247
What does Titu Andreescu have to do with Saudi Arabia?
Z K Y
The post below has been deleted. Click to close.
This post has been deleted. Click here to see post.
Medjl
757 posts
#11 • 1 Y
Y by Adventure10
ahmedAbd wrote:
What does Titu Andreescu have to do with Saudi Arabia?

he was invited to teach Saudian team
Z K Y
The post below has been deleted. Click to close.
This post has been deleted. Click here to see post.
mathcool2009
352 posts
#12 • 12 Y
Y by 62861, MathStudent2002, Makorn, niyu, sunfishho, aops29, yayups, myh2910, Adventure10, HamstPan38825, Mango247, Quidditch
I wasn't expecting this to be possible.
Degree 14 Polynomial Bash
Z K Y
The post below has been deleted. Click to close.
This post has been deleted. Click here to see post.
H.HAFEZI2000
328 posts
#13 • 3 Y
Y by 554183, Adventure10, Mango247
ahmedAbd wrote:
What does Titu Andreescu have to do with Saudi Arabia?

they probably paid for this
Z K Y
The post below has been deleted. Click to close.
This post has been deleted. Click here to see post.
Taha1381
816 posts
#14 • 1 Y
Y by Adventure10
oneplusone wrote:
It is easy to check that $a+b-c>0$ and its cyclic counterparts. Then by Holder's,
\[\sum_{cyc}{\frac{a}{(b+c-a)^2}}\sum_{cyc}a^2(b+c-a)\sum_{cyc}a^3(b+c-a)\geq (\sum_{cyc}a^2)^3=27\]

What was the motivation for this kind of Holder usage?
This post has been edited 2 times. Last edited by Taha1381, Oct 27, 2018, 2:38 PM
Z K Y
The post below has been deleted. Click to close.
This post has been deleted. Click here to see post.
WolfusA
1900 posts
#15 • 8 Y
Y by Sugiyem, PickleSauce, 508669, L567, mathscrazy, ehuseyinyigit, Adventure10, farhad.fritl
H.HAFEZI2000 wrote:
ahmedAbd wrote:
What does Titu Andreescu have to do with Saudi Arabia?
they probably paid for this
Ten oil barrels for sure.
Z K Y
The post below has been deleted. Click to close.
This post has been deleted. Click here to see post.
Math-wiz
6107 posts
#16 • 1 Y
Y by Adventure10
orl wrote:
Let $a,b$ and $c$ be positive real numbers satisfying $\min(a+b,b+c,c+a) > \sqrt{2}$ and $a^2+b^2+c^2=3.$ Prove that

\[\frac{a}{(b+c-a)^2} + \frac{b}{(c+a-b)^2} + \frac{c}{(a+b-c)^2} \geq \frac{3}{(abc)^2}.\]
Proposed by Titu Andreescu, Saudi Arabia

Everything is fine, but why Titu Andreescu, Saudi Arabia! Sorry if this is considered spam
This post has been edited 1 time. Last edited by Math-wiz, Nov 9, 2019, 4:50 AM
Z K Y
The post below has been deleted. Click to close.
This post has been deleted. Click here to see post.
solver1104
510 posts
#17 • 2 Y
Y by Adventure10, Mango247
H.HAFEZI2000 wrote:
ahmedAbd wrote:
What does Titu Andreescu have to do with Saudi Arabia?

they probably paid for this

did you not see this
Z K Y
The post below has been deleted. Click to close.
This post has been deleted. Click here to see post.
Math-wiz
6107 posts
#18 • 1 Y
Y by Adventure10
solver1104 wrote:
H.HAFEZI2000 wrote:
ahmedAbd wrote:
What does Titu Andreescu have to do with Saudi Arabia?

they probably paid for this

did you not see this

Ohh, sorry :)
Z K Y
The post below has been deleted. Click to close.
This post has been deleted. Click here to see post.
ZETA_in_olympiad
2211 posts
#20
Y by
Note that $a+b-c, b+c-a$ and $a+c-b$ are positive. Thus by Hölder: $$\sum_{\text{cyc}} a^3(b+c-a) \sum_{\text{cyc}} a^2(b+c-a) \sum_{\text{cyc}} \frac{a}{(b+c-a)^2} \geq 27.$$Since by Schur: $$3ab\geq \sum_{\text{cyc}} a^2(b+c-a) \quad \text{and} \quad abc(a+b+c)\geq \sum_{\text{cyc}} a^3(b+c-a).$$Thus the inequality of the problem holds.
Z K Y
The post below has been deleted. Click to close.
This post has been deleted. Click here to see post.
strong_boy
261 posts
#21 • 2 Y
Y by Mango247, Mango247
It is easy to see $\sum \frac{a}{(b+c-a)^2} = \sum \frac{a^6}{a^5(b+c-a)^2} $ . Now we can use $T-2$ lemma :

$$\sum \frac{a^6}{a^5(b+c-a)^2} = \sum \frac{a^6}{(a^{\frac{5}{2}}(b+c-a))^2} \geq \frac{27}{(\sum a^{\frac{5}{2}}(b+c-a))^2}$$
Now by schur it is easy to see :

$$\sum a^{\frac{5}{2}}(b+c-a))^2 \leq abc(\sum \sqrt{a})$$
Now we only need to prove :
$$\frac{27}{abc(\sum \sqrt{a})^2} \geq \frac{3}{(abc)^2}$$
Now we need to prove $\sum \sqrt{a} \leq \sqrt{3}$ .And it is easy by Holder .
Z K Y
The post below has been deleted. Click to close.
This post has been deleted. Click here to see post.
awesomeming327.
1699 posts
#22
Y by
Let $S$ denote the left hand side. We know that by Schur's Inequality,
\begin{align*} 
a^{1.5}(a-b)(a-c)+b^{1.5}(b-c)(b-a)+c^{1.5}(c-a)(c-b)\ge 0\\
a^{1.5}bc+b^{1.5}ca+c^{1.5}ab\ge a^{2.5}(b+c-a) + b^{2.5}(c+a-b)+c^{2.5}(a+b-c)
\end{align*}Since $(\sqrt{a}+\sqrt{b}+\sqrt{c})^4\le 27(a^2+b^2+c^2)=81$ by Power Mean Inequality, we have
\[3abc \ge a^{2.5}(b+c-a) + b^{2.5}(c+a-b)+c^{2.5}(a+b-c)\]By Holder's Inequality, $S(3abc)^2 \ge (a^2+b^2+c^2)^3=27$ so the desired holds.
Z K Y
The post below has been deleted. Click to close.
This post has been deleted. Click here to see post.
sqing
41782 posts
#23
Y by
Let $a,b$ and $c$ be positive real numbers satisfying $\min(a+b,b+c,c+a) > \sqrt{2}$ and $a^2+b^2+c^2=3.$ Prove or disprove
$$ \frac{a}{(b+c-a)^3} + \frac{b}{(c+a-b)^3} + \frac{c}{(a+b-c)^3} \geq \frac{3}{(abc)^3}$$
Z K Y
The post below has been deleted. Click to close.
This post has been deleted. Click here to see post.
S.Das93
707 posts
#24
Y by
Can we solve this in a similar way by showing the Triangle Inequality and then substituting the respective $a=u+v,b=v+w,c=w+u$ ?
Z K Y
The post below has been deleted. Click to close.
This post has been deleted. Click here to see post.
bin_sherlo
707 posts
#25
Y by
Since $2(a^2+b^2)\geq (a+b)^2>2$ we get $a+b>\sqrt 2>\sqrt{3-a^2-b^2}=c$ thus, $a,b,c$ satisfy the triangle inequality. Let $a=\frac{y+z}{2},b=\frac{x+z}{2},c=\frac{x+y}{2}$.
\[\sum{x}\sum{\frac{1}{x^2}}-\sum{\frac{1}{x}}=\sum{\frac{y+z}{x^2}}\overset{?}{\geq} \frac{384}{(y+z)^2(x+z)^2(x+y)^2}\]Let $x+y+z=3u, \ xy+yz+zx=3v^2,\ xyz=w^3$ where $u^2+v^2=2$.
\[3u.\frac{9v^4-6uw^3}{w^6}-\frac{3v^2}{w^3}\overset{?}{\geq} \frac{384}{(9uv^2-w^3)^2}\iff \frac{u(9v^4-6uw^3)}{w^6}\overset{?}{\geq} \frac{v^2}{w^3}+\frac{128}{(9uv^2-w^3)^2}\]Note that $v^4\geq uw^3$ holds and $u\geq 1$ since $2=u^2+v^2\leq 2u^2$.
\[\frac{u(9v^4-6uw^3)}{w^6}\geq \frac{3uv^4}{w^6}\geq \frac{3u^2}{w^3}\geq \frac{v^2}{w^3}+\frac{2u^2}{w^3}\geq \frac{v^2}{w^3}+\frac{2}{u^2v^4}\geq \frac{v^2}{w^3}+\frac{128}{(9uv^2-w^3)^2}\]As desired.$\blacksquare$
Z K Y
N Quick Reply
G
H
=
a